¿Es precisa la ley de Coulomb para cargas en movimiento?

¿Podemos usar la ley de Coulomb para calcular la fuerza entre dos cargas que no están en reposo? Si no, ¿qué fórmula se debe usar para calcular la fuerza? Lo busqué, pero no pude encontrar una respuesta clara.

Respuestas (4)

La ley de Coulomb no es exactamente cierta cuando las cargas se mueven; las fuerzas eléctricas también dependen de los movimientos de las cargas de una manera complicada. A una parte de la fuerza entre cargas en movimiento la llamamos fuerza magnética. Es realmente un aspecto de un efecto eléctrico. Por eso llamamos al tema "electromagnetismo".

Hay un principio general importante que hace posible tratar las fuerzas electromagnéticas de una forma relativamente sencilla. Encontramos, a partir de experimentos, que la fuerza que actúa sobre una carga en particular, sin importar cuántas cargas haya o cómo se muevan, depende solo de la posición de esa carga en particular, de la velocidad de la carga y de la cantidad. del cargo Podemos escribir la fuerza F en un cargo q moviéndose con una velocidad v como

F = q ( mi + v × B ) .

Llamamos mi el campo electrico y B el campo magnético en la ubicación de la carga. 1


Créditos: 1 conferencias de Feynman sobre Física-II.

Puede buscar la versión en PDF de las conferencias de Feynman sobre física. Obtendrá buenas respuestas para su pregunta en profundidad.
En el marco donde la fuente de carga es estacionaria, no hay B campo, entonces, ¿no debería estar bien la Ley de Coulomb? Si la carga de la fuente se está moviendo, entonces puedo ver que habrá un campo magnético, además de que hay que tener en cuenta la velocidad finita de transmisión del campo (velocidad de la luz). En sistemas pequeños, lo que significa casi todo lo que vemos en el día a día, el retardo puede ignorarse.
Después de reflexionar ... mi comentario anterior solo se aplica si existe un marco de inercia en el que la fuente está en reposo. Si no, la respuesta se vuelve complicada. La fuente irradiará, y tenemos campo cercano y campo lejano ... por lo que la respuesta corta al OP es "depende".
Entonces, el campo magnético que sería generado por una carga en movimiento con, por ejemplo, una velocidad de 2 m/s es insignificante. ¿Bien?

Supongamos que en algún marco de referencia S, tenemos dos partículas cargadas estacionarias , q 1 y q 2 . La fuerza experimentada por la primera partícula debido a la segunda está dada por

F 12 = k q 1 q 2 r 12 2 r ^ 12 ,

dónde k es la constante de Coulomb, q i es la carga de la partícula respectiva, r 12 es la distancia entre las dos partículas, y r ^ 12 es el punto del vector radial unitario de la partícula 2 a la partícula 1.

Como se hace comúnmente en la mayoría de los libros de texto de electromagnetismo, entonces tiene sentido introducir el vector de campo eléctrico, definido como la "fuerza por unidad de carga". En el mismo marco S, la partícula estacionaria (en la posición r = 0 ) con cargo q crea el campo eléctrico

mi ( r ) = k q r 2 r ^ ,

dónde r = | | r | | y r ^ = r | | r | | .

Con un poco de magia (es broma, todo matemáticas), es posible demostrar que esta declaración es equivalente a la ley de Gauss:

Σ mi d A = q enc ϵ 0 .

Se entiende que el campo está integrado sobre una superficie cerrada Σ y eso q enc es la cantidad de carga ubicada dentro de dicha superficie.

Resulta que la primera supuesta "definición" del campo eléctrico solo es cierta para los marcos donde la partícula está estacionaria. Para otros marcos, es necesario aplicar las transformaciones estándar de la relatividad especial. El enfoque más conveniente es reconocer que la carga es una cantidad invariable (es decir, no cambia bajo una transformada de Lorentz). Esto implica que la ley de Gauss es cierta para todos los marcos de referencia. Las únicas magnitudes que se transforman son el campo eléctrico y la superficie de integración:

Σ mi d A = Σ mi d A .

Gracias nuevamente a la relatividad especial, podemos mostrar que una partícula en movimiento debe generar un campo magnético en algunos marcos de referencia. Las expresiones finales para los campos eléctricos y magnéticos generales para un marco de referencia donde la partícula se mueve con una velocidad y aceleración arbitrarias son complicadas, pero sin embargo se pueden calcular .

Aplicando estos campos junto con la ley de fuerza de Lorentz,

F = q ( mi + v × B ) ,

podemos encontrar las trayectorias de ambas partículas. Sin embargo, estas trayectorias son bastante complicadas y deben simularse en una computadora.

TL; DR - No, en cierto sentido, la ley de Coulomb no se puede aplicar al movimiento de dos partículas cargadas que interactúan.

Como muchas personas ya han dicho en su respuesta, si te preguntas cuál es la fuerza entre dos partículas cargadas que tienen algunas velocidades en algún momento en el marco de referencia del laboratorio, entonces lo que encuentras es la fuerza de Lorentz: F = q ( mi + v × B ) .

Ahora bien, el problema es cuando intentas resolver la dinámica de tal sistema porque resulta que la fuerza de Lorentz no es recíproca , es decir, no satisface la tercera ley de Newton ( F 12 F 21 ).

Esto está muy bien explicado aquí (con animaciones y todo).

Esto ya lo descubrió Poincaré y lo desconcertó por un tiempo hasta que se dio cuenta de que la razón de esta violación era que no estábamos tomando en cuenta las variaciones del tensor de cantidad de movimiento-energía del propio campo electromagnético.

Por lo tanto, si desea resolver de verdad este problema dinámico, se trata de uno muy complejo en el que el campo electromagnético debe resolverse al mismo tiempo que las posiciones y los momentos de sus dos partículas.

Ahora, resulta que si solo está interesado en las propiedades estadísticas de un conjunto (clásico) de cargas (en equilibrio termodinámico), entonces se puede demostrar que es suficiente considerar solo la interacción de Coulomb entre cargas; entonces, en cierto modo, dependiendo de lo que quieras hacer, sí, Coulomb puede ser suficiente; de ​​lo contrario, se vuelve muy complicado.

La fuerza de Lorentz actúa sobre la carga.

F = q ( mi + v × B )

Si la carga se mueve en un campo eléctrico uniforme mi , no habrá B y la fuerza es F = q mi . En el caso de un campo eléctrico no uniforme (por ejemplo, una carga puntual), el campo eléctrico en la carga cambiará con el tiempo y, por lo tanto, según la ley de Ampere, un B será inducido. Pero por lo general (en casos no relativistas) el inducido B será despreciable.

Arriesgando una muestra de ignorancia: ¿No es cierto que en el caso de una carga puntual estacionaria, no hay campo magnético, punto. Entonces la Ley de Coulomb es válida en ese marco. En el marco que se mueve con la partícula de prueba, hay un campo magnético, por lo que la Ley de Coulomb necesita ayuda. Entonces, ¿no es la respuesta la misma que la respuesta de Benchley a la broma de Dorothy Parker sobre las chicas y los anteojos: "Depende del marco".